Finds the analytical form of the row sum of the inverse of a tridiagonal matrix.












3












$begingroup$


Let $bf A$ be an $N times N$ tri-diagonal matrix $mathbf{A}=frac{1}{{N + 1}}left( {begin{array}{*{20}{c}}
{ - 2}&1&{}&{}\
1&{ - 2}& ddots &{}\
{}& ddots & ddots &1\
{}&{}&1&{ - 2}
end{array}} right)$
. How do we find a analytical form of row sum of its inverse $mathbf{A}^{-1}$. Let $s(i)$ denote the sum of elements of the $i$th row in $mathbf{A}^{-1}$, then $s(i)=sum_{j=1}^Nmathbf{A}^{-1}(i,j)$.










share|cite|improve this question









$endgroup$

















    3












    $begingroup$


    Let $bf A$ be an $N times N$ tri-diagonal matrix $mathbf{A}=frac{1}{{N + 1}}left( {begin{array}{*{20}{c}}
    { - 2}&1&{}&{}\
    1&{ - 2}& ddots &{}\
    {}& ddots & ddots &1\
    {}&{}&1&{ - 2}
    end{array}} right)$
    . How do we find a analytical form of row sum of its inverse $mathbf{A}^{-1}$. Let $s(i)$ denote the sum of elements of the $i$th row in $mathbf{A}^{-1}$, then $s(i)=sum_{j=1}^Nmathbf{A}^{-1}(i,j)$.










    share|cite|improve this question









    $endgroup$















      3












      3








      3





      $begingroup$


      Let $bf A$ be an $N times N$ tri-diagonal matrix $mathbf{A}=frac{1}{{N + 1}}left( {begin{array}{*{20}{c}}
      { - 2}&1&{}&{}\
      1&{ - 2}& ddots &{}\
      {}& ddots & ddots &1\
      {}&{}&1&{ - 2}
      end{array}} right)$
      . How do we find a analytical form of row sum of its inverse $mathbf{A}^{-1}$. Let $s(i)$ denote the sum of elements of the $i$th row in $mathbf{A}^{-1}$, then $s(i)=sum_{j=1}^Nmathbf{A}^{-1}(i,j)$.










      share|cite|improve this question









      $endgroup$




      Let $bf A$ be an $N times N$ tri-diagonal matrix $mathbf{A}=frac{1}{{N + 1}}left( {begin{array}{*{20}{c}}
      { - 2}&1&{}&{}\
      1&{ - 2}& ddots &{}\
      {}& ddots & ddots &1\
      {}&{}&1&{ - 2}
      end{array}} right)$
      . How do we find a analytical form of row sum of its inverse $mathbf{A}^{-1}$. Let $s(i)$ denote the sum of elements of the $i$th row in $mathbf{A}^{-1}$, then $s(i)=sum_{j=1}^Nmathbf{A}^{-1}(i,j)$.







      linear-algebra






      share|cite|improve this question













      share|cite|improve this question











      share|cite|improve this question




      share|cite|improve this question










      asked Dec 4 '18 at 8:14









      TuyetTuyet

      477210




      477210






















          2 Answers
          2






          active

          oldest

          votes


















          2












          $begingroup$

          Let $s = (s_i)$ be the $Ntimes 1$ matrix of row sums. i.e. the entry at $i^{th}$ row is $s_i = s(i)$.
          Let $u$ be the $Ntimes 1$ matrix with all entries $1$. By definition of the row sums, we have
          $$s = A^{-1} u quadiffquad As = u$$
          Express everyting in terms of entries of $A$ and $s$, this is equivalent to following set of equations:
          $$begin{array}{rrrrrrl}
          - 2 s_1& + s_2&&&& = N+1\
          s_1 & -2s_2& + s_3 &&& = N+1\
          && ddots & ddots & +s_{N} &= N+1\
          &&&s_{N-1}&{ -2s_{N}} &= N+1
          end{array}tag{*1}$$

          Introduce two dummy variables $s_0 = s_{N+1} = 0$. This can be summarized as



          $$s_{k-1} - 2s_{k} + s_{k+1} = N+1quadtext{ for } k = 1,ldots, N$$



          Notice what's on the left has the form of second order finite difference. To get a constant
          on the right. $s_k$ need to be a quadratic polynomial in $k$.
          Since $s_0 = s_{N+1} = 0$, we have $s_{k} = alpha k (k - N - 1)$ for some constant $alpha$. One can fix $a$ using any equation in $(*1)$. At the end, we find $alpha = frac{N+1}{2}$ and



          $$s(k) = s_k = frac{N+1}{2}k(k - N - 1)$$






          share|cite|improve this answer











          $endgroup$





















            2












            $begingroup$

            The elements of the inverse are



            $$
            [A^{-1}]_{ij} = (N + 1) begin{cases}
            (-1)^{i + j} theta_{i - 1}phi_{j + 1}/theta_N & i < j \
            theta_{i - 1}phi_{j + 1}/theta_N & i = j tag{1} \
            (-1)^{i + j} theta_{j - 1}phi_{i + 1}/theta_N & i > j \
            end{cases}
            $$



            where $theta$ satisfy the recurrence relation



            $$
            theta_i = -2theta_{i - 1} - theta_{i - 2}
            $$



            with initial condition $theta_0 = 1$, $theta_1 = -2$ this leads to



            $$
            theta_i = (-1)^{i}(i + 1) tag{2}
            $$



            Similarly



            $$
            phi_i = -2phi_{i + 1} - phi_{i + 2}
            $$



            with conditions $phi_{N + 1} = 1$, $phi_N = -2$:



            $$
            phi_{i} = (-1)^{1 + i - N}(2 - i + N) tag{3}
            $$



            Replacing in $(1)$, you get



            begin{eqnarray}
            [A^{-1}]_{ij} &=& (N + 1) begin{cases}
            (-1)^{i + j} (-1)^{i - 1}i(-1)^{2 + j - N}(1 - j + N)/(-1)^N(N + 1) & i < j \
            (-1)^{i - 1}i(-1)^{2 + j - N}(1 - j + N)/(-1)^N(N + 1) & i = j \
            (-1)^{i + j} (-1)^{j - 1}j(-1)^{2 + i - N}(1 - i + N)/(-1)^N(N + 1) & i > j \
            end{cases} \ &=&
            -begin{cases}
            i(1 - j + N) & i < j \
            i(1 - j + N) & i = j tag{4}\
            j(1 - i + N) & i > j \
            end{cases}
            end{eqnarray}



            Now it is a matter of adding these components



            $$bbox[5px,border:2px solid blue]
            {
            s_i = sum_{j = 1}^N [A^{-1}]_{ij} = begin{cases}
            N(N - 7)(N + 1)/2 & 1 leq N leq 8 \
            4(N^2 - 6N - 7) & text{otherwise}
            end{cases}
            }
            $$






            share|cite|improve this answer











            $endgroup$













            • $begingroup$
              $N = 1, 8$ isn't a special case; the formula for $1<N<8$ still applies.
              $endgroup$
              – Display name
              Dec 4 '18 at 9:35










            • $begingroup$
              @Displayname Thanks for the suggestion. Already fixed it
              $endgroup$
              – caverac
              Dec 4 '18 at 9:37











            Your Answer





            StackExchange.ifUsing("editor", function () {
            return StackExchange.using("mathjaxEditing", function () {
            StackExchange.MarkdownEditor.creationCallbacks.add(function (editor, postfix) {
            StackExchange.mathjaxEditing.prepareWmdForMathJax(editor, postfix, [["$", "$"], ["\\(","\\)"]]);
            });
            });
            }, "mathjax-editing");

            StackExchange.ready(function() {
            var channelOptions = {
            tags: "".split(" "),
            id: "69"
            };
            initTagRenderer("".split(" "), "".split(" "), channelOptions);

            StackExchange.using("externalEditor", function() {
            // Have to fire editor after snippets, if snippets enabled
            if (StackExchange.settings.snippets.snippetsEnabled) {
            StackExchange.using("snippets", function() {
            createEditor();
            });
            }
            else {
            createEditor();
            }
            });

            function createEditor() {
            StackExchange.prepareEditor({
            heartbeatType: 'answer',
            autoActivateHeartbeat: false,
            convertImagesToLinks: true,
            noModals: true,
            showLowRepImageUploadWarning: true,
            reputationToPostImages: 10,
            bindNavPrevention: true,
            postfix: "",
            imageUploader: {
            brandingHtml: "Powered by u003ca class="icon-imgur-white" href="https://imgur.com/"u003eu003c/au003e",
            contentPolicyHtml: "User contributions licensed under u003ca href="https://creativecommons.org/licenses/by-sa/3.0/"u003ecc by-sa 3.0 with attribution requiredu003c/au003e u003ca href="https://stackoverflow.com/legal/content-policy"u003e(content policy)u003c/au003e",
            allowUrls: true
            },
            noCode: true, onDemand: true,
            discardSelector: ".discard-answer"
            ,immediatelyShowMarkdownHelp:true
            });


            }
            });














            draft saved

            draft discarded


















            StackExchange.ready(
            function () {
            StackExchange.openid.initPostLogin('.new-post-login', 'https%3a%2f%2fmath.stackexchange.com%2fquestions%2f3025277%2ffinds-the-analytical-form-of-the-row-sum-of-the-inverse-of-a-tridiagonal-matrix%23new-answer', 'question_page');
            }
            );

            Post as a guest















            Required, but never shown

























            2 Answers
            2






            active

            oldest

            votes








            2 Answers
            2






            active

            oldest

            votes









            active

            oldest

            votes






            active

            oldest

            votes









            2












            $begingroup$

            Let $s = (s_i)$ be the $Ntimes 1$ matrix of row sums. i.e. the entry at $i^{th}$ row is $s_i = s(i)$.
            Let $u$ be the $Ntimes 1$ matrix with all entries $1$. By definition of the row sums, we have
            $$s = A^{-1} u quadiffquad As = u$$
            Express everyting in terms of entries of $A$ and $s$, this is equivalent to following set of equations:
            $$begin{array}{rrrrrrl}
            - 2 s_1& + s_2&&&& = N+1\
            s_1 & -2s_2& + s_3 &&& = N+1\
            && ddots & ddots & +s_{N} &= N+1\
            &&&s_{N-1}&{ -2s_{N}} &= N+1
            end{array}tag{*1}$$

            Introduce two dummy variables $s_0 = s_{N+1} = 0$. This can be summarized as



            $$s_{k-1} - 2s_{k} + s_{k+1} = N+1quadtext{ for } k = 1,ldots, N$$



            Notice what's on the left has the form of second order finite difference. To get a constant
            on the right. $s_k$ need to be a quadratic polynomial in $k$.
            Since $s_0 = s_{N+1} = 0$, we have $s_{k} = alpha k (k - N - 1)$ for some constant $alpha$. One can fix $a$ using any equation in $(*1)$. At the end, we find $alpha = frac{N+1}{2}$ and



            $$s(k) = s_k = frac{N+1}{2}k(k - N - 1)$$






            share|cite|improve this answer











            $endgroup$


















              2












              $begingroup$

              Let $s = (s_i)$ be the $Ntimes 1$ matrix of row sums. i.e. the entry at $i^{th}$ row is $s_i = s(i)$.
              Let $u$ be the $Ntimes 1$ matrix with all entries $1$. By definition of the row sums, we have
              $$s = A^{-1} u quadiffquad As = u$$
              Express everyting in terms of entries of $A$ and $s$, this is equivalent to following set of equations:
              $$begin{array}{rrrrrrl}
              - 2 s_1& + s_2&&&& = N+1\
              s_1 & -2s_2& + s_3 &&& = N+1\
              && ddots & ddots & +s_{N} &= N+1\
              &&&s_{N-1}&{ -2s_{N}} &= N+1
              end{array}tag{*1}$$

              Introduce two dummy variables $s_0 = s_{N+1} = 0$. This can be summarized as



              $$s_{k-1} - 2s_{k} + s_{k+1} = N+1quadtext{ for } k = 1,ldots, N$$



              Notice what's on the left has the form of second order finite difference. To get a constant
              on the right. $s_k$ need to be a quadratic polynomial in $k$.
              Since $s_0 = s_{N+1} = 0$, we have $s_{k} = alpha k (k - N - 1)$ for some constant $alpha$. One can fix $a$ using any equation in $(*1)$. At the end, we find $alpha = frac{N+1}{2}$ and



              $$s(k) = s_k = frac{N+1}{2}k(k - N - 1)$$






              share|cite|improve this answer











              $endgroup$
















                2












                2








                2





                $begingroup$

                Let $s = (s_i)$ be the $Ntimes 1$ matrix of row sums. i.e. the entry at $i^{th}$ row is $s_i = s(i)$.
                Let $u$ be the $Ntimes 1$ matrix with all entries $1$. By definition of the row sums, we have
                $$s = A^{-1} u quadiffquad As = u$$
                Express everyting in terms of entries of $A$ and $s$, this is equivalent to following set of equations:
                $$begin{array}{rrrrrrl}
                - 2 s_1& + s_2&&&& = N+1\
                s_1 & -2s_2& + s_3 &&& = N+1\
                && ddots & ddots & +s_{N} &= N+1\
                &&&s_{N-1}&{ -2s_{N}} &= N+1
                end{array}tag{*1}$$

                Introduce two dummy variables $s_0 = s_{N+1} = 0$. This can be summarized as



                $$s_{k-1} - 2s_{k} + s_{k+1} = N+1quadtext{ for } k = 1,ldots, N$$



                Notice what's on the left has the form of second order finite difference. To get a constant
                on the right. $s_k$ need to be a quadratic polynomial in $k$.
                Since $s_0 = s_{N+1} = 0$, we have $s_{k} = alpha k (k - N - 1)$ for some constant $alpha$. One can fix $a$ using any equation in $(*1)$. At the end, we find $alpha = frac{N+1}{2}$ and



                $$s(k) = s_k = frac{N+1}{2}k(k - N - 1)$$






                share|cite|improve this answer











                $endgroup$



                Let $s = (s_i)$ be the $Ntimes 1$ matrix of row sums. i.e. the entry at $i^{th}$ row is $s_i = s(i)$.
                Let $u$ be the $Ntimes 1$ matrix with all entries $1$. By definition of the row sums, we have
                $$s = A^{-1} u quadiffquad As = u$$
                Express everyting in terms of entries of $A$ and $s$, this is equivalent to following set of equations:
                $$begin{array}{rrrrrrl}
                - 2 s_1& + s_2&&&& = N+1\
                s_1 & -2s_2& + s_3 &&& = N+1\
                && ddots & ddots & +s_{N} &= N+1\
                &&&s_{N-1}&{ -2s_{N}} &= N+1
                end{array}tag{*1}$$

                Introduce two dummy variables $s_0 = s_{N+1} = 0$. This can be summarized as



                $$s_{k-1} - 2s_{k} + s_{k+1} = N+1quadtext{ for } k = 1,ldots, N$$



                Notice what's on the left has the form of second order finite difference. To get a constant
                on the right. $s_k$ need to be a quadratic polynomial in $k$.
                Since $s_0 = s_{N+1} = 0$, we have $s_{k} = alpha k (k - N - 1)$ for some constant $alpha$. One can fix $a$ using any equation in $(*1)$. At the end, we find $alpha = frac{N+1}{2}$ and



                $$s(k) = s_k = frac{N+1}{2}k(k - N - 1)$$







                share|cite|improve this answer














                share|cite|improve this answer



                share|cite|improve this answer








                edited Dec 4 '18 at 9:42

























                answered Dec 4 '18 at 9:35









                achille huiachille hui

                95.7k5132258




                95.7k5132258























                    2












                    $begingroup$

                    The elements of the inverse are



                    $$
                    [A^{-1}]_{ij} = (N + 1) begin{cases}
                    (-1)^{i + j} theta_{i - 1}phi_{j + 1}/theta_N & i < j \
                    theta_{i - 1}phi_{j + 1}/theta_N & i = j tag{1} \
                    (-1)^{i + j} theta_{j - 1}phi_{i + 1}/theta_N & i > j \
                    end{cases}
                    $$



                    where $theta$ satisfy the recurrence relation



                    $$
                    theta_i = -2theta_{i - 1} - theta_{i - 2}
                    $$



                    with initial condition $theta_0 = 1$, $theta_1 = -2$ this leads to



                    $$
                    theta_i = (-1)^{i}(i + 1) tag{2}
                    $$



                    Similarly



                    $$
                    phi_i = -2phi_{i + 1} - phi_{i + 2}
                    $$



                    with conditions $phi_{N + 1} = 1$, $phi_N = -2$:



                    $$
                    phi_{i} = (-1)^{1 + i - N}(2 - i + N) tag{3}
                    $$



                    Replacing in $(1)$, you get



                    begin{eqnarray}
                    [A^{-1}]_{ij} &=& (N + 1) begin{cases}
                    (-1)^{i + j} (-1)^{i - 1}i(-1)^{2 + j - N}(1 - j + N)/(-1)^N(N + 1) & i < j \
                    (-1)^{i - 1}i(-1)^{2 + j - N}(1 - j + N)/(-1)^N(N + 1) & i = j \
                    (-1)^{i + j} (-1)^{j - 1}j(-1)^{2 + i - N}(1 - i + N)/(-1)^N(N + 1) & i > j \
                    end{cases} \ &=&
                    -begin{cases}
                    i(1 - j + N) & i < j \
                    i(1 - j + N) & i = j tag{4}\
                    j(1 - i + N) & i > j \
                    end{cases}
                    end{eqnarray}



                    Now it is a matter of adding these components



                    $$bbox[5px,border:2px solid blue]
                    {
                    s_i = sum_{j = 1}^N [A^{-1}]_{ij} = begin{cases}
                    N(N - 7)(N + 1)/2 & 1 leq N leq 8 \
                    4(N^2 - 6N - 7) & text{otherwise}
                    end{cases}
                    }
                    $$






                    share|cite|improve this answer











                    $endgroup$













                    • $begingroup$
                      $N = 1, 8$ isn't a special case; the formula for $1<N<8$ still applies.
                      $endgroup$
                      – Display name
                      Dec 4 '18 at 9:35










                    • $begingroup$
                      @Displayname Thanks for the suggestion. Already fixed it
                      $endgroup$
                      – caverac
                      Dec 4 '18 at 9:37
















                    2












                    $begingroup$

                    The elements of the inverse are



                    $$
                    [A^{-1}]_{ij} = (N + 1) begin{cases}
                    (-1)^{i + j} theta_{i - 1}phi_{j + 1}/theta_N & i < j \
                    theta_{i - 1}phi_{j + 1}/theta_N & i = j tag{1} \
                    (-1)^{i + j} theta_{j - 1}phi_{i + 1}/theta_N & i > j \
                    end{cases}
                    $$



                    where $theta$ satisfy the recurrence relation



                    $$
                    theta_i = -2theta_{i - 1} - theta_{i - 2}
                    $$



                    with initial condition $theta_0 = 1$, $theta_1 = -2$ this leads to



                    $$
                    theta_i = (-1)^{i}(i + 1) tag{2}
                    $$



                    Similarly



                    $$
                    phi_i = -2phi_{i + 1} - phi_{i + 2}
                    $$



                    with conditions $phi_{N + 1} = 1$, $phi_N = -2$:



                    $$
                    phi_{i} = (-1)^{1 + i - N}(2 - i + N) tag{3}
                    $$



                    Replacing in $(1)$, you get



                    begin{eqnarray}
                    [A^{-1}]_{ij} &=& (N + 1) begin{cases}
                    (-1)^{i + j} (-1)^{i - 1}i(-1)^{2 + j - N}(1 - j + N)/(-1)^N(N + 1) & i < j \
                    (-1)^{i - 1}i(-1)^{2 + j - N}(1 - j + N)/(-1)^N(N + 1) & i = j \
                    (-1)^{i + j} (-1)^{j - 1}j(-1)^{2 + i - N}(1 - i + N)/(-1)^N(N + 1) & i > j \
                    end{cases} \ &=&
                    -begin{cases}
                    i(1 - j + N) & i < j \
                    i(1 - j + N) & i = j tag{4}\
                    j(1 - i + N) & i > j \
                    end{cases}
                    end{eqnarray}



                    Now it is a matter of adding these components



                    $$bbox[5px,border:2px solid blue]
                    {
                    s_i = sum_{j = 1}^N [A^{-1}]_{ij} = begin{cases}
                    N(N - 7)(N + 1)/2 & 1 leq N leq 8 \
                    4(N^2 - 6N - 7) & text{otherwise}
                    end{cases}
                    }
                    $$






                    share|cite|improve this answer











                    $endgroup$













                    • $begingroup$
                      $N = 1, 8$ isn't a special case; the formula for $1<N<8$ still applies.
                      $endgroup$
                      – Display name
                      Dec 4 '18 at 9:35










                    • $begingroup$
                      @Displayname Thanks for the suggestion. Already fixed it
                      $endgroup$
                      – caverac
                      Dec 4 '18 at 9:37














                    2












                    2








                    2





                    $begingroup$

                    The elements of the inverse are



                    $$
                    [A^{-1}]_{ij} = (N + 1) begin{cases}
                    (-1)^{i + j} theta_{i - 1}phi_{j + 1}/theta_N & i < j \
                    theta_{i - 1}phi_{j + 1}/theta_N & i = j tag{1} \
                    (-1)^{i + j} theta_{j - 1}phi_{i + 1}/theta_N & i > j \
                    end{cases}
                    $$



                    where $theta$ satisfy the recurrence relation



                    $$
                    theta_i = -2theta_{i - 1} - theta_{i - 2}
                    $$



                    with initial condition $theta_0 = 1$, $theta_1 = -2$ this leads to



                    $$
                    theta_i = (-1)^{i}(i + 1) tag{2}
                    $$



                    Similarly



                    $$
                    phi_i = -2phi_{i + 1} - phi_{i + 2}
                    $$



                    with conditions $phi_{N + 1} = 1$, $phi_N = -2$:



                    $$
                    phi_{i} = (-1)^{1 + i - N}(2 - i + N) tag{3}
                    $$



                    Replacing in $(1)$, you get



                    begin{eqnarray}
                    [A^{-1}]_{ij} &=& (N + 1) begin{cases}
                    (-1)^{i + j} (-1)^{i - 1}i(-1)^{2 + j - N}(1 - j + N)/(-1)^N(N + 1) & i < j \
                    (-1)^{i - 1}i(-1)^{2 + j - N}(1 - j + N)/(-1)^N(N + 1) & i = j \
                    (-1)^{i + j} (-1)^{j - 1}j(-1)^{2 + i - N}(1 - i + N)/(-1)^N(N + 1) & i > j \
                    end{cases} \ &=&
                    -begin{cases}
                    i(1 - j + N) & i < j \
                    i(1 - j + N) & i = j tag{4}\
                    j(1 - i + N) & i > j \
                    end{cases}
                    end{eqnarray}



                    Now it is a matter of adding these components



                    $$bbox[5px,border:2px solid blue]
                    {
                    s_i = sum_{j = 1}^N [A^{-1}]_{ij} = begin{cases}
                    N(N - 7)(N + 1)/2 & 1 leq N leq 8 \
                    4(N^2 - 6N - 7) & text{otherwise}
                    end{cases}
                    }
                    $$






                    share|cite|improve this answer











                    $endgroup$



                    The elements of the inverse are



                    $$
                    [A^{-1}]_{ij} = (N + 1) begin{cases}
                    (-1)^{i + j} theta_{i - 1}phi_{j + 1}/theta_N & i < j \
                    theta_{i - 1}phi_{j + 1}/theta_N & i = j tag{1} \
                    (-1)^{i + j} theta_{j - 1}phi_{i + 1}/theta_N & i > j \
                    end{cases}
                    $$



                    where $theta$ satisfy the recurrence relation



                    $$
                    theta_i = -2theta_{i - 1} - theta_{i - 2}
                    $$



                    with initial condition $theta_0 = 1$, $theta_1 = -2$ this leads to



                    $$
                    theta_i = (-1)^{i}(i + 1) tag{2}
                    $$



                    Similarly



                    $$
                    phi_i = -2phi_{i + 1} - phi_{i + 2}
                    $$



                    with conditions $phi_{N + 1} = 1$, $phi_N = -2$:



                    $$
                    phi_{i} = (-1)^{1 + i - N}(2 - i + N) tag{3}
                    $$



                    Replacing in $(1)$, you get



                    begin{eqnarray}
                    [A^{-1}]_{ij} &=& (N + 1) begin{cases}
                    (-1)^{i + j} (-1)^{i - 1}i(-1)^{2 + j - N}(1 - j + N)/(-1)^N(N + 1) & i < j \
                    (-1)^{i - 1}i(-1)^{2 + j - N}(1 - j + N)/(-1)^N(N + 1) & i = j \
                    (-1)^{i + j} (-1)^{j - 1}j(-1)^{2 + i - N}(1 - i + N)/(-1)^N(N + 1) & i > j \
                    end{cases} \ &=&
                    -begin{cases}
                    i(1 - j + N) & i < j \
                    i(1 - j + N) & i = j tag{4}\
                    j(1 - i + N) & i > j \
                    end{cases}
                    end{eqnarray}



                    Now it is a matter of adding these components



                    $$bbox[5px,border:2px solid blue]
                    {
                    s_i = sum_{j = 1}^N [A^{-1}]_{ij} = begin{cases}
                    N(N - 7)(N + 1)/2 & 1 leq N leq 8 \
                    4(N^2 - 6N - 7) & text{otherwise}
                    end{cases}
                    }
                    $$







                    share|cite|improve this answer














                    share|cite|improve this answer



                    share|cite|improve this answer








                    edited Dec 4 '18 at 14:51

























                    answered Dec 4 '18 at 9:34









                    caveraccaverac

                    14.5k31130




                    14.5k31130












                    • $begingroup$
                      $N = 1, 8$ isn't a special case; the formula for $1<N<8$ still applies.
                      $endgroup$
                      – Display name
                      Dec 4 '18 at 9:35










                    • $begingroup$
                      @Displayname Thanks for the suggestion. Already fixed it
                      $endgroup$
                      – caverac
                      Dec 4 '18 at 9:37


















                    • $begingroup$
                      $N = 1, 8$ isn't a special case; the formula for $1<N<8$ still applies.
                      $endgroup$
                      – Display name
                      Dec 4 '18 at 9:35










                    • $begingroup$
                      @Displayname Thanks for the suggestion. Already fixed it
                      $endgroup$
                      – caverac
                      Dec 4 '18 at 9:37
















                    $begingroup$
                    $N = 1, 8$ isn't a special case; the formula for $1<N<8$ still applies.
                    $endgroup$
                    – Display name
                    Dec 4 '18 at 9:35




                    $begingroup$
                    $N = 1, 8$ isn't a special case; the formula for $1<N<8$ still applies.
                    $endgroup$
                    – Display name
                    Dec 4 '18 at 9:35












                    $begingroup$
                    @Displayname Thanks for the suggestion. Already fixed it
                    $endgroup$
                    – caverac
                    Dec 4 '18 at 9:37




                    $begingroup$
                    @Displayname Thanks for the suggestion. Already fixed it
                    $endgroup$
                    – caverac
                    Dec 4 '18 at 9:37


















                    draft saved

                    draft discarded




















































                    Thanks for contributing an answer to Mathematics Stack Exchange!


                    • Please be sure to answer the question. Provide details and share your research!

                    But avoid



                    • Asking for help, clarification, or responding to other answers.

                    • Making statements based on opinion; back them up with references or personal experience.


                    Use MathJax to format equations. MathJax reference.


                    To learn more, see our tips on writing great answers.




                    draft saved


                    draft discarded














                    StackExchange.ready(
                    function () {
                    StackExchange.openid.initPostLogin('.new-post-login', 'https%3a%2f%2fmath.stackexchange.com%2fquestions%2f3025277%2ffinds-the-analytical-form-of-the-row-sum-of-the-inverse-of-a-tridiagonal-matrix%23new-answer', 'question_page');
                    }
                    );

                    Post as a guest















                    Required, but never shown





















































                    Required, but never shown














                    Required, but never shown












                    Required, but never shown







                    Required, but never shown

































                    Required, but never shown














                    Required, but never shown












                    Required, but never shown







                    Required, but never shown







                    Popular posts from this blog

                    Le Mesnil-Réaume

                    Ida-Boy-Ed-Garten

                    web3.py web3.isConnected() returns false always